2014 dxdy logo

Научный форум dxdy

Математика, Физика, Computer Science, Machine Learning, LaTeX, Механика и Техника, Химия,
Биология и Медицина, Экономика и Финансовая Математика, Гуманитарные науки




Начать новую тему Ответить на тему На страницу Пред.  1, 2, 3, 4  След.
 
 
Сообщение27.07.2006, 21:08 
Аватара пользователя


28/06/06
138
ИСПРАВИЛ:

Руст писал(а):
Вначале надо доказать, что при k>1,y>k в разложении хотя бы одного числа входит простой множитель больше k.



Допустим это не так, тогда найдутся по крайней мере два числа
$M_1 , M_2 $ которые кратны m, где m -по условию максимальное
простое число <=k. т.е $ m=max|p_i| $, где $p_i $ -простые
и $ p_i<=k$ , тогда разность между двумя числами $D=M_1 - M_2 $ кратна m. Но поскольку числа $M_1 , M_2 $
не равны между собой и $M_1$ не делится не на одно из 2,3,..,m-1 то
оно должно делится на некоторое ПРОСТОЕ число большее k либо представлятся
ввиде $M_1=m^l$.

 Профиль  
                  
 
 
Сообщение27.07.2006, 22:15 
Заслуженный участник


09/02/06
4382
Москва
Woland писал(а):
Руст писал(а):
А вы не учли, что последнее число делится на простое число m гораздо меньше, чем k. Вот в приведённом примере k=89 а m=43.

Уважаемый Руст, доказательство взаимопростоты с числом p у меня получается из того, что я доказываю ,что существует число не
делящееся одновременно на 2,3,..,m-1. При этом совершенно не
важно на, что делятся выкинутые на том или ином этапе числа,
раз это число не делится не на одно из 2,3,..,m-1. Правда здесь
есть тонкий момент, связанный стем,что возможно на одном из
этапов, мы выкинули cамо число делящееся m.
Гипотеза: этого быть не может.

Как я и говорил контрпример достаточно большой. Минимальный из них такой:
у=14 478 292 443 583, k=89. Убедитесь, что вычеркивая по вашей методике числа из ряда y+1,y+2,y+3,...,y+89 вы последним вычеркните число делящееся на 43 - y+36. А в первом шаге вы вычеркните другое число делящееся на 43 - y+79.

 Профиль  
                  
 
 
Сообщение27.07.2006, 22:18 
Заслуженный участник


09/02/06
4382
Москва
Woland писал(а):
ИСПРАВИЛ:

Руст писал(а):
Вначале надо доказать, что при k>1,y>k в разложении хотя бы одного числа входит простой множитель больше k.



Допустим это не так, тогда найдутся по крайней мере два числа
$M_1 , M_2 $ которые кратны m, где m -по условию максимальное
простое число <=k. т.е $ m=max|p_i| $, где $p_i $ -простые
и $ p_i<=k$ , тогда разность между двумя числами $D=M_1 - M_2 $ кратна m. Но поскольку числа $M_1 , M_2 $
не равны между собой и $M_1$ не делится не на одно из 2,3,..,m-1 то
оно должно делится на некоторое ПРОСТОЕ число большее k либо представлятся
ввиде $M_1=m^l$.

Почему это число не делится ни на одно из других простых чисел. Как видно в приведённом примере это не так.

 Профиль  
                  
 
 
Сообщение27.07.2006, 22:27 
Аватара пользователя


28/06/06
138
Руст писал(а):
Woland писал(а):
ИСПРАВИЛ:

Руст писал(а):
Вначале надо доказать, что при k>1,y>k в разложении хотя бы одного числа входит простой множитель больше k.



Допустим это не так, тогда найдутся по крайней мере два числа
$M_1 , M_2 $ которые кратны m, где m -по условию максимальное
простое число <=k. т.е $ m=max|p_i| $, где $p_i $ -простые
и $ p_i<=k$ , тогда разность между двумя числами $D=M_1 - M_2 $ кратна m. Но поскольку числа $M_1 , M_2 $
не равны между собой и $M_1$ не делится не на одно из 2,3,..,m-1 то
оно должно делится на некоторое ПРОСТОЕ число большее k либо представлятся
ввиде $M_1=m^l$.

Почему это число не делится ни на одно из других простых чисел. Как видно в приведённом примере это не так.


Вы согласны ,что оно не может делится на 2 и на 3 хотябы?

 Профиль  
                  
 
 
Сообщение27.07.2006, 22:41 
Заслуженный участник


09/02/06
4382
Москва
Если речь идёт о числе, которое делится на максимальное простое число не превышающее k, то оно может делится на любые другие простые числа меньше k. Найдите из указанного промежутка, число делящееся на 89. А если речь идёт о последнем числе в вашем построении, то здесь мне пришлось найти минимальный контрпример, когда другое число, делящееся на 43 из этого интервала чётно.

 Профиль  
                  
 
 
Сообщение27.07.2006, 23:11 
Заслуженный участник
Аватара пользователя


07/03/06
1898
Москва
А тут дискуссия...
Руст писал(а):
Я неправильно высказался и исправил. А это утверждение (которое вы доказываете) вообще не верно. Пусть числа $p_1,p_2,...,p_s, P=p_1*...*p_{s-2}$ простые не превосходящие к. Тогда выбем число а: $aP=-1(mod \ p_{s-1}),aP=1(mod \ p_s),(0<a<p_{s-1}p_s)$
Тогда взяв $y=aP-b, b=p_{s-1}$ получаем, что каждое число (y+1),(y+2),...(y+2b-1) делится на простое число меньшее к. Так как 2b-1>=k при k>4 то это доказывает, что это утверждение неверно.

Но мне доказательство Руста непонятно. Ясно, что контрпример нужно искать в больших промежутках, где все числа составные. Пусть по китайской теореме мы подбираем такие $a$, что $aP=-1(mod \ p_{s-1}),aP=1(mod \ p_s),(0<a<p_{s-1}p_s)$, почему из этого должно следовать, что в произведении $(ap_1p_2..p_{s-2}-p_{s-1}+1)(ap_1p_2..p_{s-2}-p_{s-1}+2)$ $(ap_1p_2..p_{s-2}-p_{s-1}+3)...(ap_1p_2..p_{s-2}-p_{s-1}+2p_{s-1}-1)$ нет числа взаимно простого со всеми остальными. И что такое $k$ в Ваших обозначениях?
Другие рассуждения пока не смотрел, контрпример 14 478 292 443 583, k=89 проверять желание как-то не возниканет.

 Профиль  
                  
 
 
Сообщение28.07.2006, 01:04 
Аватара пользователя


28/06/06
138
Руст писал(а):
Woland писал(а):
Руст писал(а):
А вы не учли, что последнее число делится на простое число m гораздо меньше, чем k. Вот в приведённом примере k=89 а m=43.

Уважаемый Руст, доказательство взаимопростоты с числом p у меня получается из того, что я доказываю ,что существует число не
делящееся одновременно на 2,3,..,m-1. При этом совершенно не
важно на, что делятся выкинутые на том или ином этапе числа,
раз это число не делится не на одно из 2,3,..,m-1. Правда здесь
есть тонкий момент, связанный стем,что возможно на одном из
этапов, мы выкинули cамо число делящееся m.
Гипотеза: этого быть не может.

Как я и говорил контрпример достаточно большой. Минимальный из них такой:
у=14 478 292 443 583, k=89. Убедитесь, что вычеркивая по вашей методике числа из ряда y+1,y+2,y+3,...,y+89 вы последним вычеркните число делящееся на 43 - y+36. А в первом шаге вы вычеркните другое число делящееся на 43 - y+79.


Я проверил Ваш пример и действительно:
14 478 292 443 583+36=14478292443619
14 478 292 443 583+79=14478292443662
И НОД(14478292443619,14478292443662)=43 , и число
14478292443619=43*336704475433 ,причём число 336704475433 является простым.
и значит число 14478292443619 не могло быть выкинуто не на каком предыдущем шаге,
но то, что это последний шаг проверить достаточно трудно. (скажите Вы точно уверены
в том, что это число выкинется на последнем шаге, ведь весьма вероятно ,что в конце может остатся сразу несколько чисел, но не одно не имеет общих множителей простыми
меньшими k ). Кроме того я установил что в указанном Вами промежутке простых нет.
Значит гипотеза не верна.
Но не всё так плохо. По крайней мере для k<=6 это верно.

 Профиль  
                  
 
 
Сообщение28.07.2006, 02:37 
Модератор
Аватара пользователя


11/01/06
5660
См. http://dxdy.ru/viewtopic.php?p=27408#27408

 Профиль  
                  
 
 
Сообщение28.07.2006, 02:47 
Аватара пользователя


28/06/06
138
Maxal может Вы подскажете, как можно доказать,что среди k подряд идущих чисел,
найдется хотябы одно, которое делится на простое, большее k?

 Профиль  
                  
 
 
Сообщение28.07.2006, 03:12 
Аватара пользователя


28/06/06
138
maxal писал(а):
Это неверное утверждение. Минимальным контр-примером является отрезок [2184,2200], состоящий из 17 целых чисел. Более того, доказано, что для каждого существуют последовательных целых чисел, среди которых нет ни одного взаимно-простого со всеми остальными.

А доказанно ли, что среди l чисел найдётся по крайней мере одно число имеющее с
о всеми остальными лишь один общий множитель?

 Профиль  
                  
 
 
Сообщение28.07.2006, 03:22 
Модератор
Аватара пользователя


11/01/06
5660
Woland писал(а):
Maxal может Вы подскажете, как можно доказать,что среди k подряд идущих чисел,
найдется хотябы одно, которое делится на простое, большее k?

$1,2,3,\dots,k$ является контрпримером.

 Профиль  
                  
 
 
Сообщение28.07.2006, 08:26 
Заслуженный участник


09/02/06
4382
Москва
Я говорил, что при у>k=x-y в последовательности (y+1),(y+2),...,(y+k) существует число имеющее простой делитель больше к (можно даже несколько улучшить оценку больше m, который растёт примерно как k*lnk). Доказывается это простой оценкой:
$$ord_p(\frac{(y+1)(y+2)...(y+k)}{k!})\le max_{i|i\le k}(ord_p(y+i)).$$

 Профиль  
                  
 
 
Сообщение29.07.2006, 18:56 
Заслуженный участник


09/02/06
4382
Москва
Приведу вначале решение промежуточной задачи, число (y+1)(y+2)...(y+k) имеет простой делитель больше k при y>=k.
Допустим, что все простые делители не превосходят k. Тогда из приведённого неравенства получим, что
$f(y)=\frac{(y+1)(y+2)...(y+k-\pi(k))}{k!}$
не превосходит 1. Т.е. если f(y)>1, то существует простой делитель у указанного произведения больше чем k. Ясно, что f(y) многочлен от y монотонно растущий при положительных значениях y. Поэтому, если покажем, что f(k+l)>1, то при y>=k+l у указанного произведения имеется простой делитель больше k. При небольших k это неравенство не выполняется при l=0. Поэтому, пришлось гонять на компьютере до k=600. Максимальное значение при котором требуется сдвиг, для выполнения этого неравенства это k=199 l=1, максимальное значение требуемого сдвига при k=113 l=14. Так как максимальное расстояние между простыми когда они не превосходят 200 равно 14 то при 600>k>13 или выполняется неравенство или в произведении имеется простое значение, в любом случае это утверждение верно. При k<=13 максимальный сдвиг 9 и легко перебрать, что оно верно при k<=13. Для случая k>=600 (условно большие значения) для доказательства f(y=k)>1 применим аналитические методы. Вначале выясним, при каких с выполняется((2-с)k)!>(k!)^2 (эквивалентно f(k)>1 при с=pi(k)/k).
Используя формулу Стирлинга получаем:
$$(2-c)k[ln(2-c)+lnk-1]-2k(lnk-1)>2ln(\frac{k!e^k}{k^k})-ln(\frac{((2-c)k)!e^{(2-c)k}}{((2-c)k )^{(2-c)k)}})$$
Здесь правая часть O(ln k) а левая O(k). Поэтому исследуем вначале левую часть. Она убывает при 0<c<1. Так как при k>=600 pi(k)ln(k)/k<1.2 если при подставке c=1.2/ln k неравенство выполняется, то это доказывает утверждение.
При этом левая часть равна:
$k[2ln(2-\frac{1.2}{ln k})-1.2]+k\frac{1.2}{ln k}[1-ln(2-\frac{1.2}{ln k})]$
а правая часть не превосходит $(1+\frac{1}{12k})ln(2\pi k)-0.5ln(2\pi 5k/3)$
Легко проверяется, что приk>=600 уже первая член левой части положителен, второй член превосходит правую часть. Это доказывает наше утверждение при любом k.
Следствием этого утверждения является то, что при $y\le k^2+k$ наше уравнение не имет решений кроме указанных выше (тривиальных). Действительно, если y<=k(k+1), то у произведения имеется простой делитель p>=k+1. Так как p^2>x=y+k<=k^2+2k, то он входит в произведение только в один член и в первой степени.

 Профиль  
                  
 
 
Сообщение30.07.2006, 21:05 
Заслуженный участник


09/02/06
4382
Москва
По просьбе форумчан приведу краткое изложение решения.
Пусть $P=\{p_1,p_2,...,p_l\}$ множество всех простых меньших чем к. Общим делителем двух сомножителей могут быть только произведения этих чисел, меньшие чем k. Если уравнение имеет нетривиальное решение, то каждое число y+i представляется в виде произведения в первой степени некоторых из этих простых чисел и квадрата некоторого числа. Соответственно числу y+i сопоставляется некоторое подмножество S(i) множества P, а именно, состоящие из всех простых делящих y+i, степень которых нечётен. Тогда
$$y+i=a_i^2M(i), \ M(i)=\prod_{p\in S(i)} p.$$
Если для двух чисел числа M(i) и M(j) совпадают, то получаем два квадрата $|a_i^2-a_j^2|<\frac{k}{M(i)}, a_^2,a_j^2>y$, а это невозможно при доказанной ранее неравенству $y>k^2+k.$
При этом для решения каждое простое число должно входить в чётное количество подмножеств S(i).
В качестве иллюстрации k=2 два последовательных квадрата натуральных чисел (это невозможно). k=3 дает или два квадрата с расстоянием 2 или два квадрата с расстоянием 1.
k=5 имеется два простых, меньших 5 - 2 и 3. Имеется всего 4 подмножества у P, стало быть у двоих множители M(i) совпадают.
Однако, не всегда найдутся совпадающие множители. Например k=4 возможно M(1)=6,M(2)=1,M(3)=2,M(4)=3.
Введём следующую операцию умножения на множителях M(i) или сложения на S(i): По определению $M(i)*M(j)=\frac{M(i)M(j)}{gcd(M(i),M(j))^2}, S(i)+S(j)=S(i)US(j)-2S(i)*S(j)$ (сложение по модулю 2)
Тогда, всегда найдутся или две совпадающие M(i) или четыре множителя удовлетворяющие условию $M(i)*M(j)=M(s)*M(t)$, т.е произведения двух множителей после сокращения на квадрат общих множителей совпадает с соответствующей операцией для другой пары. С учётом y>k^2 это становится невозможным.
Есть и другие доказательства. Например, предположим вначале k=2m чётным и запишем уравнение в виде
$$P(t^2)=\prod_{i=1}^m (t^2-(2i-1)^2)=(2^mz)^2,t=2y+k+1>(k+1)(2k+1).$$
Выделяя отсюда полный квадрат $P(t^2)=Q(t)^2+R(t),deg R(t)\le 2[\frac{m-1}{2}].$
При этом $$Q(t)=[t^m\prod_{i=1}^m(1-\frac{(2i-2)^2}{2t^2}-\frac{(2i-1)^4}{8t^4}-... ) ]$$
Здесь из разложения по t берутся только входящие в неотрицательной степени.
И при таком решении условие t>(k+1)(2k+1) приводит к противоречию с малой разницей двух квадратов чисел.

 Профиль  
                  
 
 
Сообщение31.07.2006, 21:58 
Модератор
Аватара пользователя


11/01/06
5660
Руст писал(а):
число (y+1)(y+2)...(y+k) имеет простой делитель больше k при y>=k.

Это теорема Сильвестра-Шура, впоследствии упрощенная Эрдёшем, который доказал ее в форме:
если $n\geq 2k$, то ${n\choose k}$ имеет простой делитель $p>k$.

Фолкнер доказал более сильный результат (MR0190073 Faulkner, M. On a theorem of Sylvester and Schur. J. London Math. Soc. 41 1966 107--110):
Пусть $p_k$ - наименьшее простое большее $2k$. Тогда если $n\geq p_k$, ${n \choose k}$ имеет простой делитель $p\geq p_k$, за исключением случаев ${9 \choose 2}$ и ${10 \choose 3}$.

Затем Хэнсон доказал (MR0340162 Hanson, D. On a theorem of Sylvester and Schur. Canad. Math. Bull. 16 (1973), 195--199):
Произведение $k$ последовательных чисел $n(n+1)\cdots(n+k-1)$ больших $k$ содержит простой делитель больший $3k/2$, за исключением случаев $3\cdot 4$, $8\cdot 9$ и $6\cdot 7\cdot 8\cdot 9\cdot 10$.

 Профиль  
                  
Показать сообщения за:  Поле сортировки  
Начать новую тему Ответить на тему  [ Сообщений: 46 ]  На страницу Пред.  1, 2, 3, 4  След.

Модераторы: Модераторы Математики, Супермодераторы



Кто сейчас на конференции

Сейчас этот форум просматривают: нет зарегистрированных пользователей


Вы не можете начинать темы
Вы не можете отвечать на сообщения
Вы не можете редактировать свои сообщения
Вы не можете удалять свои сообщения
Вы не можете добавлять вложения

Найти:
Powered by phpBB © 2000, 2002, 2005, 2007 phpBB Group